More combinatorics...
by yayups, Nov 16, 2017, 6:42 AM
USAMO 2009 wrote:
Let
be a positive integer. Determine the size of the largest subset of
which does not contain three elements
,
,
(not necessarily distinct) satisfying
.






Solution
We claim the answer is
for
even and
for
odd. In both cases, we simply take the subset of all odd numbers as an equality case.
Let
be the set of positive numbers in our subset, and
be the set of negative numbers (note that
is not in our subset since
). Now, the condition is equivalent to saying that
are disjoint. Note that
![\[P+N = \{p+n : p\in P, n\in N\}.\]](//latex.artofproblemsolving.com/c/f/1/cf11e115c266431a0cb78ede848b484013f46177.png)
We now claim that
. To do this, let
and let
. Then, the following elements are in
:
This proves the claim. Now, since
are disjoint, we have that
This completes the proof for the odd case, but this is not strong enough for the even case.
We will now show that in fact
if
is even. Assume not, so then equality must hold in all the above inequalities. Therefore,
, and
. To start, since
, we must have that
, so
and
.
Say that some
. Then, since
, we must have that
. However, since
are disjoint, we must have that
, or
. Thus,
. Therefore, one sees that from the numbers
, at most
of them can be in
(note that
since
). Similarly, one can show that there are at most
elements in
. However, this implies
, which is a contradiction, since we started by assuming
. Therefore, for
even, we can have at most
elements in the subset.




Let





![\[P+N = \{p+n : p\in P, n\in N\}.\]](http://latex.artofproblemsolving.com/c/f/1/cf11e115c266431a0cb78ede848b484013f46177.png)
We now claim that




![\[x_1+y_1<x_2+y_1<\cdots<x_m+y_1<x_m+y_2<\cdots<x_m+y_n.\]](http://latex.artofproblemsolving.com/6/6/e/66e665e18efc8f829f123a2b7d0bc409b684e6c1.png)

![\[|-P|+|-N|+|P+N|\le 2n+1\implies |P|+|N|+|P|+|N|-1\le 2n+1\implies |P|+|N|\le n+1.\]](http://latex.artofproblemsolving.com/5/9/e/59e5c002887643506b9226af4a531dc0a4c65b23.png)
We will now show that in fact








Say that some


















Taiwan TST 2014 wrote:
Positive integers
(
) are arranged in a circle such that each
divides the sum of the neighbors; that is
is an integer for each
, where
,
. Prove that ![\[ 2n \le k_1 + k_2 + \dots + k_n < 3n. \]](//latex.artofproblemsolving.com/2/0/b/20b958efb33e616cd105317242b09b2f215059cd.png)



![\[ \frac{x_{i-1}+x_{i+1}}{x_i} = k_i \]](http://latex.artofproblemsolving.com/6/1/4/614c1ed73a3f522f8b49052d30f83a51990d7000.png)



![\[ 2n \le k_1 + k_2 + \dots + k_n < 3n. \]](http://latex.artofproblemsolving.com/2/0/b/20b958efb33e616cd105317242b09b2f215059cd.png)
Solution
We first establish the lower bound. Note that
where the inequality is AM-GM.
We claim that the upper bound holds for all
. We proceed by induction on
. Consider the base case
. Here,
, and
. Note that
and
, so either
and
, or
. In the first case, we get
, in the second we get
, both of which work.
Assume the upper bound holds for
. Now consider the situation for
. WLOG, let
be the maximum element. There are three cases.
Case 1:
. In this case,
. However, this clearly violates the maximality of
, so this case does not exist.
Case 2:
. In this case,
. The only way to not violate maximality of
is for
. However, one quickly sees that this implies that the whole sequence is constant, in which case the average of the
s is
, so this case is resolved.
Case 3:
. Now,
. Note that
where the inequality follows from the inductive hypothesis. Therefore, our induction, and our proof is complete.
![\[\sum_{i=1}^n k_i = \sum_{i=1}^n\left(\frac{x_{i+1}}{x_i}+\frac{x_{i-1}}{x_i}\right)\ge (2n)\sqrt[2n]{1}=2n,\]](http://latex.artofproblemsolving.com/5/c/f/5cff0052690a40fb97da0ee3f1907085749ca31b.png)
We claim that the upper bound holds for all












Assume the upper bound holds for



Case 1:



Case 2:






Case 3:



IMO 1995 wrote:
Let
be an odd prime number. How many
-element subsets
of
are there, the sum of whose elements is divisible by 





INDICATOR FUNCTIONS FOR LIFE
Define
to be the sum of the elements of some set
. Let
We want
Let
. Using the roots of unity filter to expand the indicator functions, we have that
If
, then we have that
, and otherwise,
. Thus,
Thus, the answer is
.


![\[f(x,y) = (1+xy)(1+x^2y)\cdots (1+x^{2p}y) = \sum_{A\subseteq[2p]}x^{s(A)}y^{|A|}.\]](http://latex.artofproblemsolving.com/d/2/0/d20e003b06b471a3cc111f5ba1d59d86361e7503.png)
![\[T:= -2 + \sum_{A\subseteq[2p]}\mathbf{1}_{p\mid s(A)}\mathbf{1}_{p\mid |A|}.\]](http://latex.artofproblemsolving.com/b/5/c/b5cc15fe501519c6eecfc2abd9285c4dada35283.png)

![\[2+T = \frac{1}{p^2}\sum_{j=0}^{p-1}\sum_{k=0}^{p-1}f(\omega^j,\omega^k).\]](http://latex.artofproblemsolving.com/0/9/0/090f85d03be15ce61438312ca0ff813189e6de06.png)




